RegistrierenRegistrieren   LoginLogin   FAQFAQ    SuchenSuchen   
Unendlicher Potentialtopf im klassischen Sinn
 
Neue Frage »
Antworten »
    Foren-Übersicht -> Quantenphysik
Autor Nachricht
Physiker1.1



Anmeldungsdatum: 14.11.2021
Beiträge: 19

Beitrag Physiker1.1 Verfasst am: 14. Nov 2021 23:46    Titel: Unendlicher Potentialtopf im klassischen Sinn Antworten mit Zitat

Meine Frage:
Hallo,

in der vor mir liegenden Aufgabe sollte man in den ersten Teilaufgaben den unendlich hohen Potentialtopf in der Quantenmechanik untersuchen. Sprich ein Teilchen, das mit einer Wellenfunktion beschrieben werden kann, sitzt in einem unendlich hohen Potentialtopf und kann nur diskrete Energien in diesem besitzen.

Nun soll man das gleiche Problem für einen klassischen Ball betrachten, der von einer Wand (x=0) zur anderen Wand (x=a) reflektiert wird. Dabei soll man begründen, dass die Aufenthaltswahrscheinlichkeit (Analog zur Wahrscheinlichkeitsdichte in der QM) durch

gegeben ist.
Danach soll man

über eine Periode mitteln und

berechnen.

Mir geht es jetzt hauptsächlich um den ersten Punkt, also das Begründen der Form der Aufenthaltswahrscheinlichkeit. Kann mir da jemand helfen?

Danke im Voraus!
LG

Meine Ideen:
Ich habe leider gar keinen Ansatz, wie ich das machen soll...
kleinesKorollar



Anmeldungsdatum: 21.03.2019
Beiträge: 19

Beitrag kleinesKorollar Verfasst am: 15. Nov 2021 02:37    Titel: Antworten mit Zitat

Hallo Physiker1.1,

das ist einer der größten Unterschiede zwischen klassischer Physik und Quantenmechanik. In der klassischen Physik kann man eine exakte Trajektorie eines Massepunkts beschreiben, während man in der Quantenmechanik nur Aufenthaltswahrscheinlichkeiten angeben kann.
Wenn man jetzt eine klassische Trajektorie (hier ein Ball, der zwischen den Wänden hin und her "prallt") als Wahrscheinlichkeitsdichte beschreiben will, dann muss man sich genau den obigen Gedankengang zu Gemüte führen. Ich wähle einen Punkt im Potentialtopf aus: Entweder der Ball ist an diesem Punkt, also die Wahrscheinlichkeitsdichte ist maximal, oder der Ball ist dort nicht, die Wahrscheinlichkeitsdichte ist also 0.
Wenn man diese diskrete Verteilung nun mathematisch formulieren möchte, dann verwendet man die Delta-Distribution:

Wobei eben der Beobachtungspunkt ist, an welchem geprüft wird, ob der Ball sich an diesem Punkt befindet. Und beschreibt das hin- und her- "oszillieren" des Balles.
Das Integral über die Wahrscheinlichkeitsdichte ergibt dann wieder 1, wie in der QM, da der Ball sich irgendwo im Topf befinden muss. Diese Eigenschaft wird mathematisch durch die Delta-Distribution sichergestellt:
Es gibt hierzu auch eine interessante Diskussion in einem englischsprachigen Forum (https://physics.stackexchange.com/questions/442928/probability-of-finding-a-particle-in-1d-box-from-classical-view).

Viele Grüße,
kleinesKorollar
Physiker1.1



Anmeldungsdatum: 14.11.2021
Beiträge: 19

Beitrag Physiker1.1 Verfasst am: 15. Nov 2021 16:00    Titel: Antworten mit Zitat

Hallo,

ersteinmal vielen Dank für die Antwort, das hat schonmal Vieles klarer gemacht!

Ich habe so die Gleichung begründen können.
Ich hänge allerdings noch ein bisschen an den Folgefragen. Bei der Mittelung habe ich erst folgenden Ansatz verfolgt:


Allerdings habe ich ja bei dieser Rechnung nicht über eine Periode gemittelt, sondern vielmehr den Mittelwert der Wahrscheinlichkeitsfunktion im Intervall [0,a] gebildet. Muss ich vielleicht das Integral umschreiben, um es von der Zeit t abhängen zu lassen?

Außerdem bin ich mir auch bei der Berechnung von
unsicher. Einfach weil ich nicht ganz verstehe was ich da "rechnen" soll.
Könntest Du mir da vielleicht nochmal helfen?

Viele Grüße,
Physiker1.1
TomS
Moderator


Anmeldungsdatum: 20.03.2009
Beiträge: 18082

Beitrag TomS Verfasst am: 15. Nov 2021 23:34    Titel: Antworten mit Zitat

Ich denke, du musst für Hin- und Rückweg die Bewegung





betrachten.

Dann sind die Integrale







zu berechnen.

Ich lasse dabei für den Rückweg die Zeit wieder bei t = 0 starten.

Evtl. ist noch ein Normierungsfaktor zu berücksichtigen, so dass


_________________
Niels Bohr brainwashed a whole generation of theorists into thinking that the job (interpreting quantum theory) was done 50 years ago.
Physiker1.1



Anmeldungsdatum: 14.11.2021
Beiträge: 19

Beitrag Physiker1.1 Verfasst am: 17. Nov 2021 01:21    Titel: Antworten mit Zitat

Hallo,

vielen Dank für die Antwort, das habe ich gesucht! Ich komme bei der Lösung dieses Integrals auf


Das erscheint mir auch logisch zu sein.

Viele Grüße
Physiker1.1
TomS
Moderator


Anmeldungsdatum: 20.03.2009
Beiträge: 18082

Beitrag TomS Verfasst am: 17. Nov 2021 06:36    Titel: Antworten mit Zitat

Hab‘s nicht nachgerechnet, aber ja, das klingt logisch, da die Geschwindigkeit konstant ist und daher die Zeit, die sich ein Teilchen in einem beliebigen Intervall [x,x+L] aufhält, proportional zu L und unabhängig von x und t sein muss.
_________________
Niels Bohr brainwashed a whole generation of theorists into thinking that the job (interpreting quantum theory) was done 50 years ago.
Neue Frage »
Antworten »
    Foren-Übersicht -> Quantenphysik